Thursday, December 1, 2016

calculus - Improper Integral $int_0^inftyleft(frac{tanh(x)}{x^3}-frac1{x^2cosh^2(x)}right)dx = frac{7zeta(3)}{pi^2} $



$\newcommand{\sech}{\operatorname{sech}}$
$\displaystyle \int_0^{\infty}{\left(\frac{\tanh(x)}{x^3} - \frac{\sech^2(x)}{x^2} \right)\ dx }= \frac{7\zeta(3)}{\pi^2} $



What I tried




I simplified it to -



$\displaystyle \int_0^{\infty}{\frac{\sinh(2x) - 2x}{x^3 \cosh^2(x)} \ dx}$



Then I don't know how to solve. I tried Feynman's method



$\displaystyle I(a) = \int_0^{\infty}{\frac{\sinh(ax) - ax}{x^3 \cosh^2(x)} \ dx}$



But then too it didn't help much.




I thought of replacing them with trigonometric forms and then complex number real and imaginary part but wasn't helpful much.



Please try to avoid complex analysis.


Answer



$\newcommand{\angles}[1]{\left\langle\,{#1}\,\right\rangle}
\newcommand{\braces}[1]{\left\lbrace\,{#1}\,\right\rbrace}
\newcommand{\bracks}[1]{\left\lbrack\,{#1}\,\right\rbrack}
\newcommand{\dd}{\mathrm{d}}
\newcommand{\ds}[1]{\displaystyle{#1}}

\newcommand{\expo}[1]{\,\mathrm{e}^{#1}\,}
\newcommand{\half}{{1 \over 2}}
\newcommand{\ic}{\mathrm{i}}
\newcommand{\iff}{\Longleftrightarrow}
\newcommand{\imp}{\Longrightarrow}
\newcommand{\Li}[1]{\,\mathrm{Li}_{#1}}
\newcommand{\mc}[1]{\mathcal{#1}}
\newcommand{\mrm}[1]{\mathrm{#1}}
\newcommand{\ol}[1]{\overline{#1}}
\newcommand{\pars}[1]{\left(\,{#1}\,\right)}

\newcommand{\partiald}[3][]{\frac{\partial^{#1} #2}{\partial #3^{#1}}}
\newcommand{\root}[2][]{\,\sqrt[#1]{\,{#2}\,}\,}
\newcommand{\totald}[3][]{\frac{\mathrm{d}^{#1} #2}{\mathrm{d} #3^{#1}}}
\newcommand{\ul}[1]{\underline{#1}}
\newcommand{\verts}[1]{\left\vert\,{#1}\,\right\vert}$




$\ds{\int_{0}^{\infty}
\bracks{{\tanh\pars{x} \over x^{3}} - {1 \over x^{2}\cosh^{2}\pars{x}}}\,\dd x
:\ ?}$.





\begin{align}
&\color{#f00}{\int_{0}^{\infty}
\bracks{{\tanh\pars{x} \over x^{3}} - {1 \over x^{2}\cosh^{2}\pars{x}}}\,\dd x} =
\int_{0}^{\infty}{\tanh\pars{x} - x \over x^{3}}\,\dd x +
\int_{0}^{\infty}{\tanh^{2}\pars{x} \over x^{2}}\,\dd x
\\[5mm] = &
-\,\half\int_{x\ =\ 0}^{x\ \to\ \infty}\bracks{\tanh\pars{x} - x}
\,\dd\pars{1 \over x^{2}} +

\int_{0}^{\infty}{\tanh^{2}\pars{x} \over x^{2}}\,\dd x
\\[5mm] = &\
\half\int_{0}^{\infty}{\mrm{sech}^{2}\pars{x} - 1 \over x^{2}}\,\dd x +
\int_{0}^{\infty}{\tanh^{2}\pars{x} \over x^{2}}\,\dd x =
\half\int_{0}^{\infty}{\tanh^{2}\pars{x} \over x^{2}}\,\dd x
\\[5mm] = &\
32\sum_{k = 0}^{\infty}\,\sum_{n = 0}^{\infty}\,\,\ \underbrace{%
\int_{0}^{\infty}{1 \over \bracks{\pars{2k + 1}\pi}^{\, 2} + 4x^{2}}\,
{1 \over \bracks{\pars{2n + 1}\pi}^{\, 2} + 4x^{2}}\,\dd x}
_{\ds{1 \over 8\pi^{2}\pars{2k + 1}\pars{2n + 1}\pars{k + n + 1}}}

\label{1}\tag{1}
\\[5mm] = &\
{4 \over \pi^{2}}\
\underbrace{\sum_{k = 0}^{\infty}{H_{k} + 2\ln\pars{2} \over \pars{2k + 1}^{2}}}
_{\ds{{7 \over 4}\,\zeta\pars{3}}}\label{2}\tag{2} =
\color{#f00}{{7 \over \pi^{2}}\,\zeta\pars{3}}
\end{align}




Note that





  • In \eqref{1}, we use the identity
    $\ds{{\tanh\pars{x} \over x} =
    8\sum_{j = 0}^{\infty}{1 \over \bracks{\pars{2j + 1}\pi}^{\, 2} + 4x^{2}}}$

  • The sum over $\ds{n}$, in \eqref{1}, yields a Digamma Function term
    $\ds{\Psi\pars{1 + k}}$ which explains the appearance of the Harmonic Number $\ds{H_{k} = \Psi\pars{1 + k} + \gamma}$. $\ds{\gamma}$ is the
    Euler-Mascheroni Constant.

  • $\ds{\sum_{k = 0}^{\infty}{H_{k} \over \pars{2k + 1}^{2}} =
    {1 \over 4}\bracks{7\zeta\pars{3} - \pi^{2}\ln\pars{2}}}$ is a well known result.


  • $\ds{\sum_{k = 0}^{\infty}{1 \over \pars{2k + 1}^{2}} =
    {1 \over 8}\,\pi^{2}}$.



No comments:

Post a Comment

analysis - Injection, making bijection

I have injection $f \colon A \rightarrow B$ and I want to get bijection. Can I just resting codomain to $f(A)$? I know that every function i...